常春藤专注小托福、SSAT、AP、IB、A-Level等最新考试时间、考试资讯。

您当前位置: 首页 > LSAT专区 > LSAT备考 > LSAT推理
  • LSAT分析推理之官网样题(一)

    2023-04-06 16:52:43 来源:网络
    文字:

    常春藤培训LSAT频道在此与大家分享LSAC官网上公布的LSAT分析推理样题(一),希望对大家备考LSAT分析推理有所帮助。

      The sample questions that follow are typical of the Analytical Reasoning problems you will find on the LSAT. There is a brief passage that presents a set of conditions, followed by questions about the relationships defined in the passage. While each passage among the examples here is followed by only one or two sample questions, each passage in the Analytical Reasoning section of the actual LSAT is followed by five to seven questions.

    Directions:

      Each group of questions in this section is based on a set of conditions. In answering some of the questions, it may be useful to draw a rough diagram. Choose the response that most accurately and completely answers the question and blacken the corresponding space on your answer sheet.

    Passage for Question 1

      A university library budget committee must reduce exactly five of eight areas of expenditure—G, L, M, N, P, R, S, and W—in accordance with the following conditions:

      If both G and S are reduced, W is also reduced.

      If N is reduced, neither R nor S is reduced.

      If P is reduced, L is not reduced.

      Of the three areas L, M, and R, exactly two are reduced.

      Question 1

      If both M and R are reduced, which one of the following is a pair of areas neither of which could be reduced?

      G, L

      G, N

      L, N

      L, P

      P, S

    Explanation for Question 1

      This question concerns a committee’s decision about which five of eight areas of expenditure to reduce. The question requires you to suppose that M and R are among the areas that are to be reduced, and then to determine which pair of areas could not also be among the five areas that are reduced.

      The fourth condition given in the passage on which this question is based requires that exactly two of M, R, and L are reduced. Since the question asks us to suppose that both M and R are reduced, we know that L must not be reduced:

      Reduced: M, R

      Not reduced: L

      The second condition requires that if N is reduced, neither R nor S is reduced. So N and R cannot both be reduced. Here, since R is reduced, we know that N cannot be. Thus, adding this to what we’ve determined so far, we know that L and N are a pair of areas that cannot both be reduced if both M and R are reduced:

      Reduced: M, R

      Not reduced: L, N

      Answer choice (C) is therefore the correct answer, and you are done.

      When you are taking the test, if you have determined the correct answer, there is no need to rule out the other answer choices. However, for our purposes in this section, it might be instructive to go over the incorrect answer choices. For this question, each of the incorrect answer choices can be ruled out by finding a possible outcome in which at least one of the two areas listed in that answer choice are reduced. Consider answer choice (A), which lists the pair G and L. We already know that for this question L must be one of the areas that is not reduced, so all we need to consider is whether G can be one of the areas that is reduced. Here’s one such possible outcome:

      Reduced: M, R, G, S, W

      If areas M, R, G, S, W are reduced, then the supposition for the question holds and all of the conditions in the passage are met:

      M and R are both reduced, as supposed for this question.

      Both G and S are reduced, and W is also reduced, so the first condition is satisfied.

      N is not reduced, so the second condition is not relevant.

      P is not reduced, so the third condition is not relevant.

      Exactly two of L, M, and R are reduced, so the fourth condition is satisfied.

      Thus, since G could be reduced without violating the conditions, answer choice (A) can be ruled out. Furthermore, since G appears in the pair listed in answer choice (B), we can also see that (B) is incorrect.

      Now let’s consider answer choice (D), which lists the pair L and P. We already know that for this question L must be one of the areas that is not reduced, so all we need to consider is whether P can be one of the areas that is reduced. Here’s one such possible outcome:

      Reduced: M, R, P, S, W

      If areas M, R, P, S, and W are reduced, then the supposition for the question holds and all of the conditions in the passage are met:

      M and R are both reduced, as supposed for this question.

      G is not reduced, so the first condition is not relevant.

      N is not reduced, so the second condition is not relevant.

      P is reduced and L is not reduced, so the third condition is satisfied.

      Exactly two of L, M, and R are reduced, so the fourth condition is satisfied.

      Thus, since P could be reduced without violating the conditions, answer choice (D) can be ruled out. Furthermore, since P appears in the pair listed in answer choice (E), we can also see that answer choice (E) is incorrect.

      This question was of moderate difficulty, based on the number of test takers who answered it correctly when it appeared on the LSAT. The most commonly selected incorrect answer choice was response (E).


本文地址:https://www.ivyeducation.cn/lsattuili/7326.html

以上便是“LSAT分析推理之官网样题(一)”的全部内容,更多最新资讯请关注常春藤教育。

标签:lsat分析推理lsat备考

- 声明 -

1、由于考试政策等各方面情况的不断调整与变化,本网站所提供的考试信息仅供参考,请以权威部门公布的正式信息为准。

2、本文内容、图片由互联网用户自发贡献,该文观点仅代表作者本人,本站仅提供信息存储空间服务,不拥有所有权,不承担相关法律责任。如您发现本站有涉嫌抄袭/侵权/违法违规的内容信息,请发送邮件至shanghai60@cedca.cn举报,一经查实,本站将立即删除。